Resistencia equivalente en circuito de escalera [cerrado]

Me he topado con una pregunta dada que realmente me cuesta resolver. Básicamente, necesito encontrar una resistencia equivalente en alguna forma de configuración de "escalera". Donde la cadena es una secuencia infinita de resistencias.

escalera

Realmente no tengo una buena idea de cómo encontrar esta resistencia equivalente. Probando la antigua regla de paralelo y resistencias en serie, llegué a una fórmula muy complicada:

R + 1 1 R + 1 R + 1 1 R + . . . .

Sin embargo, sé que la solución debería ser mucho más simple.

Ahora intenté usar la regla del bucle de Kirchhoff . Lo que establece que la diferencia de potencia en un circuito cerrado debe ser 0. Nombrar las "corrientes potenciales" entre AB I 1 , entre BC (a través de la resistencia única) I 2 y la corriente "de B a la derecha" I 3 Teniendo en cuenta el ciclo que contiene BC y el resto de la estructura, esto se reescribe en:

I 1 = I 2 + I 3
I 3 R I 2 R mi q = 0

El problema es que 2 variables, 2 funciones realmente no me acercan a una respuesta :(. ¿Qué me estoy perdiendo?

Relacionado: physics.stackexchange.com/q/10615/2451 y enlaces allí.

Respuestas (3)

SUGERENCIA :

Darse cuenta de

R mi q = R + 1 1 R + 1 R mi q

A partir de esto, de hecho, siga la respuesta rápidamente. Ahora para entender por qué se le permite hacer tal declaración en este caso. Supongo que se debe al hecho de que la secuencia tiene un tamaño infinito; como tal, "reescribir" la primera área ABCD no afecta R mi q (En otras palabras R mi q es el mismo incluso después de eliminar una ocurrencia). ¿Es esta la suposición correcta para hacer en tal pregunta?
Sí, es correcto siempre que la fracción continua converja ( R mi q es finito). Y converge porque es desviado por primera R .
+1: Muy buen argumento, y pensar que estaba sentado aquí tratando de resolver las relaciones de recurrencia :)

Puedes dividir el circuito en n etapas, la primera etapa a la derecha mostrará una resistencia r_0=2R y luego tendrás que moverte hacia la izquierda. En el norte ª etapa tendrás resistencia desconocida r norte y en el ( norte + 1 ) ª etapa tendrás resistencia:

r norte + 1 = R + R r norte R + r norte ,
siendo la serie de R con el paralelo entre R y r norte . Esta es una sucesión recursiva que, si converge a un límite L , se mostrará L = r norte = r norte + 1 cuando norte tiende al infinito. Sustituyendo obtendrás:
L = R + R L R + L
proporcionando la ecuación de segundo grado L 2 R L R 2 = 0 . El límite deseado L es pues el R mi q que estas buscando:
R mi q = L = R 2 ( 1 + 5 )

Hola Antonio, deberías usar el marcado mathjax: meta.math.stackexchange.com/q/5020

es muy sencillo, sigue estos pasos

1) a medida que el circuito tiende al infinito, considere que la resistencia equivalente es R(e) 2) considere las resistencias en paralelo, es decir, BC y el resto a la derecha de BC. estos están en serie con AB 3)ahora la resistencia total a la derecha de BC será igual a la resistencia efectiva del circuito total R(e), ya que el circuito tiende al infinito _nota: esto es posible solo en series infinitas en escalera. 4) la resistencia efectiva será R(e)= r + {R(e)*r}/{R(e)+r} 5) al resolver esto obtendrás una ecuación cuadrática que tiene un +ve y un - ve raíz. La raíz +ve da la respuesta. nota: aquí {R(e)*r}/{R(e)+r} da la resistencia efectiva de las resistencias en paralelo como se mencionó anteriormente.